LSAT and Law School Admissions Forum

Get expert LSAT preparation and law school admissions advice from PowerScore Test Preparation.

 emigini
  • Posts: 6
  • Joined: Aug 13, 2014
|
#16097
Hello,

I had narrowed the answers down to A and D and incorrectly chose D. A confused me because I wasn't sure if it meant that *only* the untreated plots would be effectively controlled. If that were the case, why would the treated plots not be controlled? I might be misinterpreting the passage, but I thought the predatory mites would simply adjust to lower levels of prey mite reproduction (and, despite these lower reproductive levels, survive long enough to control the population once prey reproduction increased).

Thank you!
 Ron Gore
PowerScore Staff
  • PowerScore Staff
  • Posts: 220
  • Joined: May 15, 2013
|
#16149
Hi Emigini,

Thanks for your question. You've guessed the reason for your confusion: you've misinterpreted answer choice (A), which says that "In both treated and untreated plots inhabited by both Typhlodromus and cyclamen mites, the latter would have been effectively controlled." You interpreted the word "latter" to refer to "untreated plots." However, the context of both the passage and the answer choice indicate that the word "latter" refers to the "cyclamen mites." It is the pests that the pesticide would control, not the plot.

In answer choice (D), it does not appear that the Typhlodromus mite populations would eventually increase, because the cyclamen populations would be kept low and the Typhlodromus population increases as the cyclamen population increases.

Please let me know if I can be of further assistance.

Thanks!

Ron
 brcibake
  • Posts: 55
  • Joined: Jul 19, 2017
|
#39231
Hello!
What type of question is this? It was hypothetical and confused me. I picked D but I think it is primarily due to misunderstanding the passage and the question.
 Adam Tyson
PowerScore Staff
  • PowerScore Staff
  • Posts: 5271
  • Joined: Apr 14, 2011
|
#39462
It's a Must Be True question, brcibake, testing both your understanding of what the passage said about the adaptability of the predator ("Its population can increase as rapidly as that of its prey") and your careful reading of each answer choice. In both RC and LR you will sometimes get these "suppose" question stems, posing a hypothetical that essentially adds a new premise to the stimulus. Treat it just that way - act as if the new info was already included in the passage or stimulus and then determine what inference you could draw from that.
 swong1267
  • Posts: 24
  • Joined: Nov 25, 2017
|
#42394
Hi,

I picked A, but I wasn't very comfortable with it. How could we know that cyclamen mites would be effectively controlled?
 Adam Tyson
PowerScore Staff
  • PowerScore Staff
  • Posts: 5271
  • Joined: Apr 14, 2011
|
#42400
We know from the text, swong, that cyclamen mites are effectively controlled in untreated plots that have Typhlodromus mites. That's the gist of the passage overall - that certain pesticides are counter-productive because they reduce the predator and allow the prey to thrive, and that when left alone the prey do not rise to damaging levels.

We also know that T mite populations grow and shrink in response to the size of the C mite populations. As C reproduces, T suddenly kicks into gear and starts producing and keeps the Cs in check. When C slows down, T gets quiet and survives in small numbers on honeydew (yum!). So, if in our hypothetical situation we did something that slowed down the C population growth, we can infer that the T population growth would slow to match it, breeding just enough to keep C from getting out of hand and doing substantial damage to the strawberries. It all flows from the facts of the passage!
 CRicksy
  • Posts: 2
  • Joined: Oct 19, 2018
|
#60150
Hello!
When completing this question, I eliminated A. My thinking was that the new pesticide X, if it's acting like the parathion, would kill the T mite in the treated plot, which would then allow the C mite population to continue growing and reaching those damaging levels (as mentioned in E). Is my thinking wrong mostly because the question stem says that pesticide X has "no other direct effect on C mites or T mites"? I must admit that I just now focused in on that part of the question when writing this question.

When I completed the question, I mostly focused on the passage, which says that the parathion eliminated the predators, which led me to E.

Thanks.
 Brook Miscoski
PowerScore Staff
  • PowerScore Staff
  • Posts: 418
  • Joined: Sep 13, 2018
|
#60290
Cricksy,

The issue is that pesticide X is the opposite of parathion, not the same as parathion.

The passage explained that parathion killed the T-mite but not the C-mite. The passage uses the parathion experiment to show that the T-mite is effective at controlling the C-mite by itself and that eliminating the T-mite is harmful to crops.

The question stem explains that pesticide X is the opposite--it helps eliminate the C-mite but has no effect on the T-mite. So if we re-ran the experiment using pesticide X, the results would be different. The pesticide X plants would have reduced C-mites from both the pesticide and the T-mites, and the no-pesticide plants would have reduced C-mites just like in the original experiment, and C-mites would be reduced in both groups. That is answer (A).
 ataraxia10
  • Posts: 46
  • Joined: Oct 04, 2018
|
#63091
I chose E but in hindsight, I suppose it would be too presumptuous to explain such detailed response to pesticide X. A is simpler and more plausible given the information from the passage? Is that why A is correct but not E?
 ataraxia10
  • Posts: 46
  • Joined: Oct 04, 2018
|
#63092
I chose E but in hindsight, I suppose it would be too presumptuous to explain such detailed response to pesticide X. A is simpler and more plausible given the information from the passage? Is that why A is correct but not E?

Get the most out of your LSAT Prep Plus subscription.

Analyze and track your performance with our Testing and Analytics Package.